Find the sum of the first 12 terms of the sequence 512, 256, 128, … This is infinite series notation, the answer is NOT 896...

Find The Sum Of The First 12 Terms Of The Sequence 512, 256, 128, This Is Infinite Series Notation, The

Answers

Answer 1

Answer:

1023.75

Step-by-step explanation:

The sum of a geometric sequence is

sum = a( 1 - r^n) / (1-r)

where a is the first term  r is the common ratio and  r^n is the nth term

We need to find the common ratio

r = 256/512 = 1/2

sum = 512 ( 1 - 1/2^12) / ( 1-1/2)

       =512( 1-.000244141) / (.5)

       =512(.999755859) /.5

     =1023.75

Answer 2

Answer:

1023.75

Step-by-step explanation:

sum = a( 1 - r^n) / (1-r)

a1 = 512

n = 12

r = 256 / 512 = 1/2

                              512 (1 - 1/2¹²)

therefore.. sum =  ------------------ = 1023.75

                                   1 - 1/2


Related Questions

line and passes through C -2,0 in the 1, -3) Quetion of the line in standard form

Answers

Answer:

[tex]\huge\boxed{x+y=-2}[/tex]

Step-by-step explanation:

The standard form of an equation of a line:

[tex]Ax+By=C[/tex]

The point-slope form of an equation of a line:

[tex]y-y_1=m(x-x_1)[/tex]

where

[tex]m=\dfrac{y_2-y_1}{x_2-x_1}[/tex]

We have two points (-2, 0) and (1, -3).

Substitute:

[tex]x_1=-2;\ y_1=0;\ x_2=1;\ y_2=-3[/tex]

[tex]m=\dfrac{-3-0}{1-(-2)}=\dfrac{-3}{1+2}=\dfrac{-3}{3}=-1\\\\y-0=-1(x-(-2))\\\\y=-(x+2)[/tex]

[tex]y=-x-2[/tex]         add x to both sides

[tex]x+y=-2[/tex]

The residents of a city voted on whether to raise property taxes. The ratio of yes votes to no votes was 6 to 5 . If there were 4545 no votes, what was the total number of votes?

Answers

Answer:

The total number of votes= 9999

Step-by-step explanation:

The ratio of vote specifically the ratio of yes to no vote in a city vote is 6 to 5.

There is a total of 4545 no votes.

Yes/no = 6/5

Yes= no(6/5)

Yes= 4545(6/5)

Yes= 5454

The total number of yes votes are 5454.

The total number of votes= yes votes+ no votes

The total number of votes= 5454+4545

The total number of votes= 9999

F
13
5
H
12
G
se
Find mZH to the nearest degree.
67
O 18
O 45
O 23

Answers

Answer:

∠ H ≈ 23°

Step-by-step explanation:

Using the tangent ratio in the right triangle

tan H = [tex]\frac{opposite}{adjacent}[/tex] = [tex]\frac{FG}{HG}[/tex] = [tex]\frac{5}{12}[/tex] , thus

∠ H = [tex]tan^{-1}[/tex] ( [tex]\frac{5}{12}[/tex] ) ≈ 23° ( to the nearest degree )

Please answer this correctly without making mistakes

Answers

Answer:

5/12

Step-by-step explanation:

3/4-1/3=

9/12-4/12=

5/12

logx - logx-1^2=2log(x-1)

Answers

Answer:

x is approximately 2.220744

Step-by-step explanation:

This can be simplified a little using properties of logarithms, and then solve it by graphing:

[tex]log(x)-log(x-1)^2=2\,log(x-1)\\log(x)-2\,log(x-1)=2\,log(x-1)\\log(x)=4\,log(x-1)[/tex]

So we use a graphing tool to find the intersection point of the graph of [tex]log(x)[/tex], and the graph of [tex]4\,log(x-1)[/tex]

Please see attached image for the graph and solution.

The value of x is approximately 2.220744

Answer:

  x = 2.32011574011

Step-by-step explanation:

The problem with your original equation is that it is a long way of saying ...

  log(x) -log(x) -1 = 2log(x-1)

  0 -1 = 2log(x-1)

which has the solution ...

  -1/2 = log(x -1)

  1/√10 = x -1

  x = 1 + 1/√10 ≈ 1.3162278

__

We have asked for clarification, and what we got was ...

  [tex]\log{(x)}-\log{(x-1^2)}=2\log{(x-1)}[/tex]

which, again, is a long way of saying ...

  [tex]\log{(x)}-\log{(x-1)}=2\log{(x-1)}[/tex]

The other reasonable interpretation of your 'clarified' equation is ...

  [tex]\log{(x)}-\log{((x-1)^2)}=2\log{(x-1)}[/tex]

which you already have an answer to. You have declared that a "misconception."

So, we are left with the interpretation that the equation you want a solution to is ...

  [tex]\log{(x)}-\log{(x-1)}=2\log{(x-1)}[/tex]

_____

When solving these graphically, I like to write the equation as a function whose zero(s) we're trying to find. For this, when we subtract the right side, we get ...

  [tex]f(x)=\log{(x)}-3\log{(x-1)}[/tex]

A graphing calculator shows that f(x) = 0 when ...

  x ≈ 2.32011574011

__

If you don't like my interpretation, check out the second attachment. It has your x-1² as the argument of the middle term. You can see that the calculator interpreted that the same way I did (as required by the order of operations).

A random sample of 10 single mothers was drawn from a Obstetrics Clinic. Their ages are as follows: 22 17 27 20 23 19 24 18 19 24 We want to determine at the 5% significance level that the population mean is not equal to 20. What is the rejection region?

Answers

Answer:

0.09

Step-by-step explanation:

Let x = ages of mother

x  :  22   17    27    20    23     19      24    18    19    24

N = 10

Mean = ∑x/N = 218/10 = 21.8

Difference in mean = 21.8 - 20 = 1.8

If significance level = 5% or 0.05

∴ Rejection region = 1.8 X 0.05 = 0.09

Un taxímetro inicia con 50 unidades y el banderazo o arranque es de $4500, las unidades comienzan a cambiar p0r cada kilometros recorrido. La función lineal que representa esta situación es y = 50x +4500 donde y representa el precio que cuesta la carrera y x la distancia recorrida en kilómetros. a) ¿ Cuanto cuesta una carrera si la distancia recorrida fue de 23 kilómetros?

Answers

Answer: $5650

Step-by-step explanation:

El precio de la carrera es:

y = ($50/km)*x + $4500.

Donde x representa la cantidad recorrida en Km.

Ahora se nos pregunta:

¿ Cuanto cuesta una carrera si la distancia recorrida fue de 23 kilómetros?

Para esto, debemos reemplazar la variable en la equacion por 23km:

x = 23km

y = ($50/km)*23km + $4500 = $5650

There are $400$ pages in Sheila's favorite book. The average number of words per page in the book is $300$. If she types at an average rate of $40$ words per minute, how many hours will it take to type the $400$ pages of the book?

Answers

Answer:

50hours

Step-by-step explanation:

Given that there are 400 pages in Sheila's favorite book.

The average number of words per page in the book is 300

She types an average rate of 40words per minute.

So to type 400pages of the book

Total number of words in the pages = 400×300 = 120000 words

Typing rate : 40words ------- 1minute

120000 words ----------- x minutes

Hence we have 40 × X mins = 120000 × 1min

Make X the subject

40X = 120000minutes

X = 120000/40

X = 3000minutes

Since 60minutes = 1hour

3000minutes = 3000minutes/60

= 50hours

Hence it took her 50hours to type 400pages

Solution:

The total number of words in the book is 400 x 300. Sheila types at a rate of 40 words per minute, or 40 x 60 words per hour. The number of hours it takes her is equal to the number of words divided by her rate of typing, or 400x300/40x60 = 50 hours.

Lydia drives from city a to city b to transport goods. her return speed is 3 times her departure speed and she takes 40 minutes less on her return trip. how long did her departure trip take?

Answers

Answer:

1 hour

Step-by-step explanation:

Hello, let's say that her departure trip takes t in minutes, as her return speed is 3 times her departure speed, she took t/3 for the return and we know that this 40 minutes less, so we can write.

t/3=t-40

We can multiply by 3

t = 3t -40*3 = 3t - 120

This is equivalent to

3t -120 = t

We subtract t

2t-120 = 0

2t = 120

We divide by 2

t = 120/2 = 60

So this is 60 minutes = 1 hour.

Thank you.

In your own words, define Quadratic Equation. How many solutions does a Quadratic Equation have?

Answers

Answer: an equation that has one term which is nameless and squared also no term which gets raised to higher power.

Step-by-step explanation:

Given v(x) = g(x) (3/2*x^4 + 4x – 1), find v'(2).​

Answers

Answer:

Step-by-step explanation:

Given that v(x) = g(x)×(3/2*x^4+4x-1)

Let's find V'(2)

V(x) is a product of two functions

● V'(x) = g'(x)×(3/2*x^4+4x-1)+ g(x) ×(3/2*x^4+4x-1)

We are interested in V'(2) so we will replace x by 2 in the expression above.

g'(2) can be deduced from the graph.

● g'(2) is equal to the slope of the tangent line in 2.

● let m be that slope .

● g'(2) = m =>g'(2) = rise /run

● g'(2) = 2/1 =2

We've run 1 square to the right and rised 2 squares up to reach g(2)

g(2) is -1 as shown in the graph.

■■■■■■■■■■■■■■■■■■■■■■■■■■

Let's derivate the second function.

Let h(x) be that function

● h(x) = 3/2*x^4 +4x-1

● h'(x) = 3/2*4*x^3 + 4

● h'(x) = 6x^3 +4

Let's calculate h'(2)

● h'(2) = 6 × 2^3 + 4

● h'(2) = 52

Let's calculate h(2)

●h(2) = 3/2*2^4 + 4×2 -1

●h(2)= 31

■■■■■■■■■■■■■■■■■■■■■■■■■■

Replace now everything with its value to find V'(2)

● V'(2) = g'(2)×h(2) + g(2)× h'(2)

● V'(2)= 2×31 + (-1)×52

●V'(2) = 61 -52

●V'(2)= 9

i will give brainliest and 5 stars if you help ASAP​

Answers

Answer:

[tex] Area = 240 m^2 [/tex]

Step-by-step explanation:

The area of the right triangle above = [tex] \frac{1}{2}*base*height [/tex].

Where,

base = 16 m

height = 30 m

Plug in the above values into the area formula:

[tex] Area = \frac{1}{2}*16*30 [/tex]

[tex] Area = 8*30 [/tex]

[tex] Area = 240 m^2 [/tex]

Refer to the attachment for solution.

The points (-6,-4) and (3,5) are the endpoints of the diameter of a circle. Find the length of the radius of the circle.
The length of the radius is a
(Round to the nearest hundredth as needed.)

Answers

Answer:

40.5

Step-by-step explanation:

diameter^2 = (3 +6)^2 + (5+4)^2

or, d^2 = 9^2 + 9^2

or, d^2 = 81 +81

or,d^2 =162

or d=√ 162

• d= 81

then radius = d/2

r = 81/2

•r= 40.5 ans

if a salesman has a base salary of 35,000 per year makes 5% commission on each sales ,how much must he do in sales to make a total of 75,000 for the year

Answers

The answer will be 3750

He must do a 8,00,000 sales to make total of 75000 for the year.

For salesman base salary = 35000, Salary to be atained is 75000. Having commission of 5% on every sales. Sales to be determine so the salesman attained 75000 for year.

What is arithmetic?

In mathematics it deals with numbers of operations according to the statements.

Here, according to the statement.
Let x be sales,
35,000 + 5%x = 75,000
0.05x = 75000-35000
x = 40000/0.05
x = 8,00,000

Thus, he must do a 8,00,000 sales to make total of 75000 for the year

Learn more about arithmetic here:

brainly.com/question/14753192

#SPJ2

A diameter that is perpendicular to a chord bisects the chord. True False

Answers

Answer:

[tex]\Large \boxed{\sf True}[/tex]

Step-by-step explanation:

[tex]\sf A \ diameter \ that \ is \ perpendicular \ to \ a \ chord \ bisects \ the \ chord.[/tex]

Answer:

True!!

I just did the assignment and got it right

Diana paints 150 fence posts and chuck paints 130 fence posts. Diana paints 10 more fence posts than chuck. How many fence posts does chuck paint per hour?

Answers

Complete Question

The  complete question is shown on the first uploaded image

Answer:

Step-by-step explanation:

From the question we are told that

  The  relationship is  [tex]\frac{150 }{d} = \frac{130}{c}[/tex]

   The number of fence post painted by chuck is  [tex]l = 130[/tex]

    The number of fence post painted by Diana  is [tex]k = 150[/tex]

    can paint 10 fences more than chuck  so let say the  of fence painted in an hour by chuck is [tex]g[/tex]

     Then  the number of fence post painted by Diana in one hour is

          [tex]f = g+ 10[/tex]

 So

       [tex]\frac{150 }{ g + 10 } = \frac{130}{g}[/tex]

       [tex]130 g + 1300 = 150g[/tex]

        [tex]g = 65 \ m[/tex]

Solve systems of equations 15 points NOT CLICKBAIT!!! -6y+11y= -36 -4y+7x= -24

Answers

Answer:

x = -264/35

y = -36/5

Step-by-step explanation:

-6y + 11y = -36

-4y + 7x = -24

Solve for y in the first equation.

-6y + 11y = -36

Combine like terms.

5y = -36

Divide both sides by 5.

y = -36/5

Plug y as -36/5 in the second equation and solve for x.

-4(-36/5) + 7x = -24

Expand brackets.

144/5 + 7x = -24

Subtract 144/5 from both sides.

7x = -264/5

Divide both sides by 7.

x = -264/35

Answer: -264/35

Step-by-step explanation:

i did my work on a calculator

A special mixed-nut blend at a store cost $1.35 per lb, and in 2010 the blend cost $1.83 per lb. Let y represent the cost of a pound of the mixed-nut blend x years after 2005. Use a linear equation model to estimate the cost of a pound of the mixed-nut blend in 2007.

Answers

Answer:

y = $1.542 per lb

Step-by-step explanation:

given data

mixed-nut blend store cost  2005 = $1.35 per lb

blend cost in 2010 = $1.83 per lb

solution

we consider here y = cost of a pound

and x year = after 2005

we will use here linear equation model

so

[tex]\frac{y - 1.35}{1.83-1.35} = \frac{x-10}{5 - 0}[/tex]    .........................1

solve it we get

5y - 6.75 = .48 x

so

at 2007 year here x wil be 2

so

[tex]y = \frac{0.48 \times 2 + 6.75}{5}[/tex]  

solve it we get

y = $1.542 per lb

Factor.
x2 – 5x - 36

(x - 9)(x + 4)
(x - 12)(x + 3)
(x + 9)(x - 4)
(x + 12)(x - 3)

Answers

Answer:

The answer is option A

Step-by-step explanation:

x² - 5x - 36

To factor the expression rewrite -5x as a difference

That's

x² + 4x - 9x - 36

Factor out x from the expression

x( x + 4) - 9x - 36

Factor out -9 from the expression

x( x + 4) - 9( x+ 4)

Factor out x + 4 from the expression

The final answer is

( x - 9)( x + 4)

Hope this helps you

Answer:

[tex] \boxed{(x - 9) \: (x + 4) }[/tex]

Option A is the correct option.-

Step-by-step explanation:

( See the attached picture )

Hope I helped!

Best regards!

find the derivative of f(x)=3x^2✓x​

Answers

Answer:

  [tex]f'(x)=\dfrac{15x\sqrt{x}}{2}[/tex]

Step-by-step explanation:

The power rule applies.

  d(x^n)/dx = nx^(n-1)

__

  [tex]f(x)=3x^2\sqrt{x}=3x^{\frac{5}{2}}\\\\f'(x)=3(\frac{5}{2})x^{\frac{3}{2}}\\\\\boxed{f'(x)=\dfrac{15x\sqrt{x}}{2}}[/tex]

What is 1/3 of 675 is left

Answers

1/3 of 675 is 225
I hope that helps

What Number is equivalent to 4^3
A. 7
B. 12
O C. 64
D. 81

Answers

Answer:

C

Step-by-step explanation:

4³ means 4 multiplied by itself 3 times, that is

4 × 4 × 4

= 16 × 4

= 64 → C

Combine like terms to simplify the expression: 2/5k - 3/5 +1/10k

Answers

━━━━━━━☆☆━━━━━━━

▹ Answer

1/2k - 3/5

▹ Step-by-Step Explanation

2/5k - 3/5 + 1/10k

Collect like terms:

2/5k + 1/10k = 1/2

Final Answer:

1/2k - 3/5

Hope this helps!

CloutAnswers ❁

━━━━━━━☆☆━━━━━━━

Answer:

1/2k - 3/5

Step-by-step explanation:

Hey there!

Well the only fraction needed to combine are,

2/5 and 1/10.

To add them we need to make 2/5 have a denominator of 10.

To do that we multiply 5 by 2.

5*2 = 10

What happens to the denominator happens to the denominator.

2*2 = 4

Fraction - 4/10

4/10 + 1/10 = 5/10

5/10

simplified

1/2

1/2k - 3/5

Hope this helps :)

Michael records the height of 1000 people. This data is a normal distribution and the sample mean was 0.75. Identify the margin of error for this data set.

Answers

Answer:

0.0284

Step-by-step explanation:

The formula for calculating the Margin of error of a dataset is expressed as;

Margin of error = [tex]Z*\sqrt{\frac{p(1-p)}{n} } \\\\[/tex] where;

Z is the z-score of 95% confidence interval = 1.96

p is the sample proportion/mean = 0.75

n is the sample size = total number of people = 1000

Note that when the confidence interval is not given, it is always safe to use 95% confidence.

Substituting this values into the formula we have;

[tex]ME = 1.96*\sqrt{\frac{0.7(1-0.7)}{1000} } \\\\ME = 1.96*\sqrt{\frac{0.7(0.3)}{1000} } \\\\ME = 1.96*\sqrt{0.00021} } \\\\ME = 1.96*0.01449\\\\ME = 0.0284[/tex]

Hence the margin error for the dataset is 0.0284

Given that
[tex]\sqrt{2p-7}=3[/tex]
and
[tex]7\sqrt{3q-1}=2[/tex]
Evaluate
[tex]p + {q}^{2} [/tex]​

Answers

Answer:

Below

Step-by-step explanation:

The two given expressions are:

● √(2p-7) = 3

● 7√(3q-1) = 2

We are told to evaluate p+q^2

To do that let's find the values of p and q^2

■■■■■■■■■■■■■■■■■■■■■■■■■■

Let's start with p.

● √(2p-7) = 3

Square both sides

● (2p-7) = 3^2

● 2p-7 = 9

Add 7 to both sides

● 2p-7+7 = 9+7

● 2p = 16

Divide both sides by 2

● 2p/2 = 16/2

● p = 8

So the value of p is 8

■■■■■■■■■■■■■■■■■■■■■■■■■■

Let's find the value of q^2

● 7√(3q-1) = 2

Square both sides

● 7^2 × (3q-1) = 2^2

● 49 × (3q-1) = 4

● 49 × 3q - 49 × 1 = 4

● 147q - 49 = 4

Add 49 to both sides

● 147q -49 +49 = 4+49

● 147q = 53

Divide both sides by 147

● 147q/147 = 53/147

● q = 53/ 147

Square both sides

● q^2 = 53^2 / 147^2

● q^2 = 2809/21609

■■■■■■■■■■■■■■■■■■■■■■■■■

● p+q^2 = 8 +(2809/21609)

● p+q^2 = (2809 + 8×21609)/21609

● p+q^2 = 175681 / 21609

● p + q^2 = 8.129

Round it to the nearest unit

● p+ q^2 = 8

Find the mean, variance, and standard deviation of the binomial distribution with the given values of n and p. n = 50 p = 0.2

Answers

Answer:

The mean, variance, and standard deviation of the binomial distribution are 10, 8, and 2.83 respectively.

Step-by-step explanation:

We have to find the mean, variance, and standard deviation of the binomial distribution with the given values of n and p, i.e; n = 50 p = 0.2.

Let X = binomial random variable

So, X ~ Binom(n = 50, p = 0.2)

Now, the mean of the binomial distribution is given by;

         Mean of X, E(X) = n [tex]\times[/tex] p

                                    = 50 [tex]\times[/tex] 0.2 = 10

Now, the variance of the binomial distribution is given by;

        Variance of X, V(X) = n [tex]\times[/tex] p [tex]\times[/tex] (1 - p)

                                         = 50 [tex]\times[/tex] 0.2 [tex]\times[/tex] (1 - 0.2)

                                         = 10 [tex]\times[/tex] 0.8 = 8

Also, the standard deviation of the binomial distribution is given by;

        Standard deviation of X, S.D.(X) = [tex]\sqrt{\text{n} \times \text{p} \times (1 - \text{p})}[/tex]

                                                              = [tex]\sqrt{\text{50} \times \text{0.2} \times (1 - \text{0.2})}[/tex]

                                                              = [tex]\sqrt{8}[/tex] = 2.83

Manuel says that he can solve the equation 3n = 21 by multiplying both sides by ⅓. Explain why this is correct.

Answers

Step-by-step explanation:

はい、両側を削除して、3を掛けて7にします

Step-by-step explanation:

Given:

3n = 21

if we multiply both sides by 1/3, we will get:

3n = 21

3n x (1/3)= 21 x (1/3)

3n/3 = 21/3

n = 21/3

n = 7

Hence we can indeed solve for n by multiplying both sides by (1/3)

What are the solution(s) of the quadratic equation 98 - x2 = 0?
x = +27
Ox= +63
x = +7/2
no real solution

Answers

Answer:

±7 sqrt(2) = x

Step-by-step explanation:

98 - x^2 = 0

Add x^2 to each side

98 =x^2

Take the square root of each side

±sqrt(98) = sqrt(x^2)

±sqrt(49*2) = x

±7 sqrt(2) = x

Answer:

[tex]\huge \boxed{{x = \pm 7\sqrt{2} }}[/tex]

Step-by-step explanation:

[tex]98-x^2 =0[/tex]

[tex]\sf Add \ x^2 \ to \ both \ sides.[/tex]

[tex]98=x^2[/tex]

[tex]\sf Take \ the \ square \ root \ of \ both \ sides.[/tex]

[tex]\pm \sqrt{98} =x[/tex]

[tex]\sf Simplify \ radical.[/tex]

[tex]\pm \sqrt{49} \sqrt{2} =x[/tex]

[tex]\pm 7\sqrt{2} =x[/tex]

[tex]\sf Switch \ sides.[/tex]

[tex]x= \pm 7\sqrt{2}[/tex]

Complete the square to make a perfect square trinomial. Then, write the result as a binomial squared. n^2+5/2n

Answers

Answer:  [tex]\bigg(n+\dfrac{5}{4}\bigg)^2[/tex]

Step-by-step explanation:

[tex]n^2+\dfrac{5}{2}n+\underline{\qquad}\\\\\\n^2+\dfrac{5}{2}n+\bigg(\dfrac{5}{2\cdot 2}\bigg)^2\\\\\\n^2+\dfrac{5}{2}n+\bigg(\dfrac{5}{4}\bigg)^2\\\\\\=\bigg(n+\dfrac{5}{4}\bigg)^2[/tex]

Write the equation of the line that passes through (−2, 6) and (2, 14) in slope-intercept form. (2 points)

Answers

Answer:

[tex]y = 4x + 14[/tex]

Step-by-step explanation:

Equation of a line is y = mx + c

where

m is the slope

c is the y intercept

To find the equation we must first find the slope of the line

Slope of the line using points (−2, 6) and (2, 14) is

[tex]m = \frac{14 - 6}{2 + 2} = \frac{8}{2} = 4[/tex]

Now we use the slope and any of the points to find the equation of the line.

Equation of the line using point ( - 2, 6) and slope 4 is

[tex]y - 6 = 4(x + 2) \\ y - 6 = 4x + 8 \\ y = 4x + 8 + 6[/tex]

We have the final answer as

[tex]y = 4x + 14[/tex]

Hope this helps you

Other Questions
Type the correct answer in the box. Use numerals instead of words.Find the area of this shape.4 cm2 cm4 cm4 cm5.75 cmThe area of the shape issquare centimeters. Which of the following was NOT an objective of the US bombing campaigns? A. Destroy the defenses and industry of the North B. Persuade the North to stop supporting the Vietcong C. Kill and remove the main leader of North Vietnam D. Boost morale of South Vietnamese soldiers PV = nRT. If P = 1 atm, V = 5.0 liter, R = 0.0821 L.atm/mol.K, and T = 293 K; what is the value of n? find the sum of partial fraction for x^4/(x-1)(x-4) For song, issuing from its birth-place, after fulfillment, wandering Reck'd or unreck'd, duly with love returns' a) Why are the last lines put within brackets? b) Reck'd or unreck'd', what does this phrase mean? c) Where does the song return? Farming, grazing of livestock, mining, and drilling combined account for: Over 1/2 of deforestation About 1/3 of deforestation 3/4 of deforestation All of deforestation La ___________ est afectando a las grandes capitales del mundo. You are aware that your neighbor trades stocks based on confidential information he overhears at his workplace. This information is not available to the general public. This neighbor continually brags to you about the profits he earns on these trades. Given this, you would tend to argue that the financial markets are at best _____ form efficient. Witch statement correctly compares the analysis and conclusion section of a lab report To measure Monarch butterfly migration,scientist tag and release 100 butterflies inKansas and Missouri before traveling toMexico. While in Mexico, the samescientist captured another 100 butterfliesof which 15 are tagged. Based on thisinformation, how many butterflies wouldyou predict start in Kansas and Missouriand migrate to Mexico? 1 person = 40 min = 1 bag, 4 person = ? min = 1575 bags The population of fruit flies tripled every day and the population today is 200 fruit flies. a) Write an equation modeling the growth in the population of fruit flies.b) What was the population of fruit flies 3 days ago? Metals and energy currency futures contracts are actively traded on Group of answer choices propane. gold. All of the options are correct. gold and silver. silver. This expression represents the average cost per game, in dollars, at a bowling alley, where n represents the number of games:3n+7/nWhat is the average cost per game if James bowls 4 games? This table represents a quadratic function.What is the value of a in the functions equation?A. -1/2B. 1/2C. 2D. 1 Consider a small country that is closed to trade, so its net exports are equal to zero. The following equations describe the economy of this country in billions of dollars, where C is consumption, D is disposable income, I is investment, and G is government purchases: C=40+0.9DIG=80I=201. Assume that this economy initially has a fixed tax and that net taxes (taxes minus transfer payments are $100 billion. Disposable income is then (Y100), where Y is the real GDP. Aggregate output demanded is $500 billion (True/False).2. Suppose the government decides to increase spending by $10 billion without raising taxes. Because the expenditure multiplier is 10 (True/False), this will increase the economy's aggregate output demanded by $100 billion (True/False).3. Now, suppose that the government switches to an income tax, which is the type of variable tax, of 20%. Because consumers retain only 80% of each additional dollar of income, disposable income is now 0.80Y. In this case, the economy's aggregate output demanded is $500 billion (True/False).4. Given an income tax of 20%, the expenditure multiplier is approximately 3.6 (True/False). Therefore, if the government decides to increase spending by $10 billion without raising tax rates, this would increase the economy's aggregate output demanded by approximately $36 billion (True/False).5. A $10 billion increase in government purchases will have a larger effect on output under a fixed tax of $100 billion (True/False). All else equal, which of the following is most likely to increase a company's retained earnings breakpoint? a. An increase in the company's dividend payout ratio b. both factors will lead to an increase in the retained earnings breakpoint. c. A decrease in the fraction of equity used in the company's target capital structure The role of the ___________ branch of the autonomic nervous system mediates control of organ processes when the body is essentially ______. Johnny and a robot standing 5 melo (units of length) apart (in a flat area) on theplanet Rote. They spot a flying object hovering in the sky at the same time. If theangle of elevation from Johnny to the flying object is 29, and the angle of elevationfrom the robot to the flying object is 42, find the distance from the flying object tothe ground. For this problem, assume that the heights of Johnny and the robot areneligible. [8 marks] please help me guys please find the value of 3x